help honestly MARKING BRAINLIESt

Help Honestly MARKING BRAINLIESt

Answers

Answer 1

Answer:

See below ~

Step-by-step explanation:

a) Forming the equation :

⇒ We start with a number, x

⇒ Now we double it so : 2(x) = 2x

⇒ Then we add 11 to it : 2x + (11) = 2x + 11

⇒ It is equal to 25 : 2x + 11 = 25

b) Solving the equation :

Subtract 11 from both sides :

⇒ 2x + 11 - 11 = 25 - 11

⇒ 2x = 14

Divide 2 on both sides :

⇒ 2x/2 = 14/2

x = 7

Answer 2

Let unknown number be x

Double it

2x

Add eleven

2x+11

You got 25

2x+11=25

Lets solve the equation

2x=25-112x=14x=7


Related Questions

state one example of 2 numbers where the LCM of the 2 numbers is the product of those numbers. explain your answers​

Answers

Answer:

See below

Step-by-step explanation:

3 and 5     have a LCM of 15    which is  3 * 5

32 out of 40 kids said they prefer summer over winter. 25 out of 32 adults said they prefer summer. Did a larger percentage of kids or adults prefer summer.

Answers

Answer:

Larger percentage of kids

Step-by-step explanation:

Kids: 32/40 = 4/5

Adults: 25/32 = 5/8

4/5 > 5/8

Katrina drinks 0.5 gallons of water per day. Which expression shows how to find the number of cups of water she drinks in a week?

There are 16 cups in a gallon.

Answers

Answer:

D

Hope this helps!

a-3/7 devided by 3-a/21

Answers

The answer is pretty simple but I put it in a picture below

Probability and Two-Way Tables

Answers

Answer:

the answer are A and Y , B and X.

 A and X are two independent events  , Option A is the correct answer.

When is Probability ?

Probability is the likelihood of an event to happen

A two way table is given

The two events will be independent when

If A is the first event and B is the second event , then

P(A) = P(A|B)

Here the P(A) = 30/100 = 0.3

P(B) = 20/100 = 0.2

P(A|X) = 15/50 = 0.3

P(A) = P(A|X)

Therefore A and X are two independent events , Option A is the correct answer.

To know more about Probability

https://brainly.com/question/11234923

#SPJ1

what 1 - 2/9 - 1/3 - 1/6 =

Answers

Answer:

5/18

Step-by-step explanation:

The LCD of 9, 3, and 6 is 18.

1 - 2/9 - 1/3 - 1/6 =

= 18/18 - 4/18 - 6/18 - 3/18

= 5/18

Answer: 5/18

Step-by-step explanation:

1/1 - 2/9 - 1/3 - 1/6

18 - 4 - 6 - 3/18

5/18

The lengths represented by BE, EC, and CD on the diagram were determined to be 1800 feet, 200 feet, and 500 feet,
What is the length, in feet, across the lake?
Fill in the blank by entering only a number as your answer.

Answers

The length of lake is 4500 feet.

What is Triangle?

A triangle is a closed shape with 3 angles, 3 sides, and 3 vertices. A triangle with three vertices P, Q, and R is represented as △PQR.

Here, In ΔABE and ΔDCE

          ∠ABE = ∠DCE         { each 90⁰ }

          ∠AEB = ∠CED         { vertically opposite angle}

By AA similarity, we get

          ΔABE ≈ ΔDCE

Now, AB/CD = BE/CE

       AB/500 = 1800/200

       AB/500 = 9

        AB = 9 X 500

       AB = 4500 feet.

Thus, the length of lake is 4500 feet.

Learn more about Triangle from:

https://brainly.com/question/2773823

#SPJ1

EMERGENCY!! 25 POINTS!!
Make a table for the following data.

a) English, Maths, Urdu, Science, English, Urdu, Science, Maths, English, Computer, Maths, Urdu, Science, Maths, Computer, English.

Answers

Using tally marks make a table of the given subjects English (4), Urdu (3), Maths (4), Science (3) and Computer (2).

The given data are English, Maths, Urdu, Science, English, Urdu, Science, Maths, English, Computer, Maths, Urdu, Science, Maths, Computer, and English.

What is a data table?

A table is an arrangement of information or data, typically in rows and columns, or possibly in a more complex structure. Tables are widely used in communication, research, and data analysis.

The data table for the given data is given below:

Thus, using tally marks make a table of the given subjects English (4), Urdu (3), Maths (4), Science (3) and Computer (2).

To learn more about the data table visit:

https://brainly.com/question/17084863.

#SPJ1

Answer:

huh

Step-by-step explanation:

Here's an equation of a line:
y - 4x = 9
Which of the following is the equation of a line
that is parallel?
Click on the correct answer.
-4y+ x = 18
y + 4x = 4
2y - 8x = 3
4y + x = 18

Answers

Answer:

[tex]2y - 8x = 3[/tex]

Step-by-step explanation:

[tex]y - 4x = 9[/tex]

[tex]2y - 8x = 18[/tex]

So 2y - 8x = 3 is parallel to y - 4x = 9.

Help please asap points an brainlist

Answers

(X,y) —> slope of line
M —> given point on the line
(X1,y1)—> any point on line

Which of the following statements is false?

Answers

In quadrilateral PQRS, ∠P and ∠S are opposite angles.

Solve for x using quadratic formula :
abx^2 + (b^2 - ac)x - bc = 0​

Answers

Answer:

[tex]\boxed{\sf x = \dfrac{c}{b} \quad or \quad \dfrac{-b}{a}}[/tex]

Explanation:

Given expression: (ab)x^2 + (b^2 - ac)x + (-bc) = 0

Here given:

a = abb = b² - acc = -bc

Apply quadratic formula:

[tex]\sf x = \dfrac{ -b \pm \sqrt{b^2 - 4ac}}{2a} \quad when \ ax^2 + bx + c = 0[/tex]

Insert values:

[tex]\sf x = \dfrac{-(b^2 - ac) \pm \sqrt{(b^2 -ac)^2-4(ab)(-bc)} }{2(ab)}[/tex]

[tex]\sf x = \dfrac{-b^2 + ac \pm \sqrt{\left(b^2-ac\right)^2+4abbc} }{2ab}[/tex]

[tex]\sf x = \dfrac{-b^2 + ac \pm \sqrt{b^4+2b^2ac+a^2c^2} }{2ab}[/tex]

[tex]\sf x = \dfrac{-b^2 + ac \pm \sqrt{\left(b^2+ac\right)^2} }{2ab}[/tex]

[tex]\sf x = \dfrac{-b^2 + ac \pm( b^2+ac )}{2ab}[/tex]

[tex]\sf x = \dfrac{-b^2 + ac +( b^2+ac )}{2ab} \quad or \quad \dfrac{-b^2 + ac -( b^2+ac )}{2ab}[/tex]

[tex]\sf x = \dfrac{2ac}{2ab} \quad or \quad \dfrac{-2b^2}{2ab}[/tex]

[tex]\sf x = \dfrac{c}{b} \quad or \quad \dfrac{-b}{a}[/tex]

(ab)x^2 + (b^2 - ac)x -bc= 0

a = abb = b² - acc = -bc

Apply quadratic formula:

[tex]\sf x = \dfrac{ -b \pm \sqrt{b^2 - 4ac}}{2a} [/tex]

[tex]\\ \sf\Rrightarrow x = \dfrac{-(b^2 - ac) \pm \sqrt{(b^2 -ac)^2-4(ab)(-bc)} }{2(ab)}[/tex]

[tex]\\ \sf\Rrightarrow x= \dfrac{-b^2 + ac \pm \sqrt{\left(b^2-ac\right)^2+4abbc} }{2ab}[/tex]

[tex]\\ \sf\Rrightarrow x = \dfrac{-b^2 + ac \pm \sqrt{b^4+2b^2ac+a^2c^2} }{2ab}[/tex]

[tex]\\ \sf\Rrightarrow x = \dfrac{-b^2 + ac \pm \sqrt{\left(b^2+ac\right)^2} }{2ab}[/tex]

[tex]\\ \sf\Rrightarrow x = \dfrac{-b^2 + ac \pm( b^2+ac )}{2ab}[/tex]

[tex]\\ \sf\Rrightarrow x = \dfrac{-b^2 + ac +( b^2+ac )}{2ab} \quad or \quad \dfrac{-b^2 + ac -( b^2+ac )}{2ab}[/tex]

[tex]\\ \sf\Rrightarrow x = \dfrac{2ac}{2ab} \quad or \quad \dfrac{-2b^2}{2ab}[/tex]

[tex]\\ \sf\Rrightarrow x = \dfrac{c}{b} \quad or \quad \dfrac{-b}{a}[/tex]

A sign on a roadway at the top of a mountain indicates that for the next 4 miles, the grade is 10.5°. Find the change in elevation over that distance for a car descending the mountain. Round to the nearest hundredth.

Answers

the change in elevation is -0.73 miles (the negative sign is because the new elevation is smaller than the initial one).

How to find the change in elevation?

To do this, we can think on the situation as a right triangle, where the hypotenuse is 4 miles, the angle that we look at measures 10.5°, and the change in elevation (let's call it x) will be the opposite cathetus to that angle.

Then we can use the relation:

Sin(a) = (opposite cathetus)/(hypotenuse)

Replacing what we know, we get:

sin(10.5°) = x/4mi

x = sin(10.5°)*4mi = 0.73mi

So the change in elevation is -0.73 miles (the negative sign is because the new elevation is smaller than the initial one).

If you want to learn more about right triangles:

https://brainly.com/question/2217700

#SPJ1

Can someone help me please and thxxx

Answers

Answer:

C

Step-by-step explanation:

cos 30=12/x

[tex]x = 8 \sqrt{3} [/tex]

cos60=y/x

[tex]y =4 \sqrt{3} [/tex]

Answer:

x = [tex]8\sqrt{3}[/tex]

y = [tex]4\sqrt{3}[/tex]

Step-by-step explanation:

• tan 60° = 12/ y         [tan θ = opposite/ adjacent]

y = 12/ tan 60°  

y = [tex]4\sqrt{3}[/tex]

• x² = y² + 12²             [Pythagoras's theorem]

x² = ( [tex]4\sqrt{3}[/tex] )² + 144

x = [tex]\sqrt{(4\sqrt{3})^{2} \space\ + \space\ 144}[/tex]

x = [tex]8\sqrt{3}[/tex]

please help guys how do i order fractions for least to greatest and greatest to least itll nean so much to me for my finals tmrw

Answers

Answer:

If the denominators are equal for all of the fractions, you can simply look at the numerator and order them. If the denominators aren't equal, you'll have to find the lcm (lowest common multiple), for example, if we have the fractions 3/6 and 1/4, we would find the lcm of 6 and 4, which is 12 and multiply the numerators accordingly. So the fractions would then become 6/12 and 3/12, this makes it easier for you to figure out their order from least to greatest.

Goodluck for your finals :)

1 − 2 + 3 −... + 99
Please solve!

Answers

Answer:

50

Step-by-step explanation:

So you can think of this by grouping it like this:

(1-2) + (3-4) + (5-6) + ... + (97-98) + 99

which is equal to: (-1) + (-1) + (-1)... + (-1) + 99

(each group is equal to -1, and 99 won't have a pair since it's the last one)

then, find how many groups of -1 there are:

the groups start at 1 and end at 98, but there are two in each group, so 98/2 = 49. this means there are 49 groups.

so now, you know that there are 49 -1s, so 49 * (-1) = -49.

finally, you can't forget the extra 99 that didn't have a pair, so -49 + 99 = 50.

05.02 MC)
Angie is working on solving the exponential equation 34^x= 23; however, she is not quite sure where to start. Using complete sentences, describe to Angie how to solve this equation.

Answers

Step-by-step explanation:

First she will start by mutiplying the two numbers.

Use the grouping method to factor x³ + x² + 3x+3.
A. (x²+1)(x+3)
B. x(x+3)(x + 1)
C. (x + 1)(x+3)
D. (x + 1)(x2+3)

Answers

Answer:

D. [tex](x + 1)(x^2 + 3)[/tex]

Step-by-step explanation:

Hello!

We can group the first two terms and the last two terms.

Factor by Grouping[tex]x^2 + x^2 + 3x + 3[/tex][tex]x^2(x + 1) + 3(x + 1)[/tex][tex](x^2 + 3)(x + 1)[/tex]

Factoring by grouping is the process of breaking down larger polynomials to smaller ones to factor. We can then combine like factors.

In the second step, we can see that we can rewrite [tex]x^3 + x^2[/tex] as [tex]x^2(x + 1)[/tex], as both the two terms share a common factor of [tex]x^2[/tex]. We can pull out [tex]x^2[/tex] from that expression. Similarly, [tex]3x[/tex] and [tex]3[/tex] share a common factor of [tex]3[/tex], so we can pull that out.

Given the following data points, calculate the curve of best fit. show all steps.

Answers

Based on the calculations, the equation for the curve of best fit is equal to y = -30.17x + 14.49.

How to calculate the curve of best fit?

From the table of data points, we have the following:

∑x = 16∑y = 50.9∑xy = 24.6∑x² = 35

Mathematically, the standard equation of a straight line is given by:

y = ax + b       ....equation 1.

Thus, the equations that can be used to model the given data points are:

∑y = na + b∑x             ....equation 2.

∑xy = a∑x + b∑x²       ....equation 3.

Substituting the parameters into the equations, we have;

50.9 = 6a + 16b             ....equation 4.

24.6 = 16a + 35b       ....equation 5.

Solving eqn. 5 and 6 simultaneously, we have:

a = -30.17.b = 14.49.

Substituting the value of a and b into eqn. 1, we have;

y = ax + b

y = -30.17x + 14.49.

Therefore, the equation for the curve of best fit is equal to y = -30.17x + 14.49.

Read more on curve of best fit here: brainly.com/question/9366563

#SPJ1

Click an item in the list or group of pictures at the bottom of the problem and, holding the button down, drag it into the correct position in the answer box. Release your mouse button when the item is place. If you change your mind, drag the item to the trashcan. Click the trashcan to clear all your answers.
One leg of a right triangle is 2 inches and the hypotenuse is 6 inches.

Find the area of the triangle.

_[tex]\sqrt{x} \\[/tex]

Answers

We can see that the area of the triangle is: 5.65in².

What is a triangle?

A triangle is actually known to be a shape that has three sides, three angles and three vertices. It's also known as a plane shape.

In order to find the area of the triangle, we will use Pythagorean Theorem: c² = a² + b².

Where c = hypotenuse = 6in.

b = 2in.

6² = a² + 2²

36 = a² + 4

a² = 36 - 4 = 32

a = √32

Area of the triangle = ½ × base × height.

Where base = √32

height = 2

Thus: A = ½ × √32 × 2

A = √32 = 5.65in².

Learn more about area of triangle on https://brainly.com/question/17335144

#SPJ1

what is the length of the base? help me please thank u ;)​

Answers

Answer:   26

=============================================================

Explanation:

a = 26 and b = 26 are the congruent legs

angle C = 60 degrees is between sides 'a' and b

c = unknown base which is opposite angle C

We'll use the law of cosines to find this missing side.

c^2 = a^2 + b^2 - 2*a*b*cos(C)

c^2 = 26^2 + 26^2 - 2*26*26*cos(60)

c^2 = 676

c = sqrt(676)

c = 26

It turns out that all three sides are the same length (26), which means this isosceles triangle is really equilateral. Consequently, it means all three interior angles are 60 degrees each.

---------------

Here's another way to see why we have an equilateral triangle.

The vertex angle is 60 degrees. Let x be the measure of each base angle. Those two base angles add to the 60 degrees to get 180

x+x+60 = 180

2x+60 = 180

2x = 180-60

2x = 120

x = 120/2

x = 60

Each base angle is 60 degrees, so all three angles are 60 degrees. This points us to the triangle being equilateral and we can say all three sides are 26 units long.

If we didn't have an equilateral triangle, then we'd have no choice but to use the law of cosines mentioned earlier.

[tex]\quad \huge \quad \quad \boxed{ \tt \:Answer }[/tex]

[tex]\qquad \tt \rightarrow \:base \:\:side = 26 \:\: units [/tex]

____________________________________

[tex] \large \tt Solution \: : [/tex]

As the two sides of the triangle are equal, the corresponding angles opposite to the the sides are equal as well.

[tex] \textsf{let each of those angles measure ' x ' } [/tex]

[tex]\qquad \tt \rightarrow \:x + x + 60 = 180[/tex]

[ sum of all interior angles of a triangle ]

[tex]\qquad \tt \rightarrow \:2x + 60 = 180[/tex]

[tex]\qquad \tt \rightarrow \:2x = 180 - 60[/tex]

[tex]\qquad \tt \rightarrow \:2x = 120[/tex]

[tex]\qquad \tt \rightarrow \:x = \cfrac{120}{2} [/tex]

[tex]\qquad \tt \rightarrow \:x = 60 \degree[/tex]

Therefore, all angles of the triangle are equal. that being the case we can conclude that it's an equilateral triangle.

henceforth, all its sides are equal to one another.

[tex]\qquad \tt \rightarrow \:base \: \: side = \: \: 26 \: \: units[/tex]

The question is the image below.

Answers

Answer:

Hence - 7 is the answer

Pls mark me brainliest plss

Which function is graphed?

Answers

Answer:

science type of food group : fibre

the bearing of two points x and y from z are 45° and 135° respectively . if |zx|=8cm and |zy|=6cm, find |xy|.

Answers

Answer:

[tex]|{\sf XY}| = 10\; {\rm cm}[/tex].

Step-by-step explanation:

Refer to the diagram attached. The dashed segment attached to [tex]\!{\sf Z}[/tex] points to the north. Rotating this segment clockwise with point [tex]{\sf Z}\!\![/tex] as the fixed center of rotation would eventually align this segment with the one between point [tex]\!\!{\sf Z}[/tex] and point [tex]\!\!{\sf X}[/tex]. The bearing of point [tex]{\sf X}[/tex] from point [tex]{\sf Z}[/tex] is the size of the angle between these two line segments when measured in the clockwise direction.

Subtract the bearing of [tex]{\sf Y}[/tex] from [tex]{\sf Z}[/tex] from the bearing of [tex]{\sf X}[/tex] from [tex]{\sf Z}[/tex] to find the measure of the angle [tex]\angle {\sf YZX}[/tex]:

[tex]\begin{aligned}\angle {\sf YZX} &= 135^{\circ} - 45^{\circ} \\ &= 90^{\circ}\end{aligned}[/tex].

Thus, triangle [tex]\triangle {\sf YZX}[/tex] is a right triangle ([tex]90^{\circ}[/tex]) with segment [tex]{\sf YX}[/tex] as the hypotenuse. It is given that [tex]|{\sf XZ}| = 6\; {\rm cm}[/tex] whereas [tex]|{\sf ZY}| = 6\; {\rm cm}[/tex]. Thus, by Pythagorean's Theorem:

[tex]\begin{aligned}|{\sf ZY}| &= \sqrt{|{\sf ZX}|^{2} + |{\sf ZY}|^{2}} \\ &= \sqrt{(8\; {\rm cm})^{2} + (6\; {\rm cm})^{2}} \\ &= 10\; {\rm cm}\end{aligned}[/tex].

Need help please. Math and I'll give 30 points. Please and thank you. :D

And have a great day! :)

Answers

Answer:

h = 17.5 m

Step-by-step explanation:

We have 2 similar right triangles as models of the situation

one has legs with George's height and shadow

the other is the height h of the tree and length of it's shadow

the corresponding sides of the 2 triangles are in proportion, that is

[tex]\frac{6.25}{h}[/tex] = [tex]\frac{10}{28}[/tex] ( cross- multiply )

10h = 175 ( divide both sides by 10 )

h = 17.5 m

Use the accompanying table to answer the questions regarding money ratios.
(a) What is the capital a 40-year-old should have with an income of $32,000?

(b) How much should a 35-year-old be saving for retirement with an income of $86,000?

(c) How much in education debt should a 30-year-old have with an income of $59,000?


The 40-year-old should have $___in capital.
(Simplify your answer)

(b) The 35-year-old should be saving $ for retirement.

(Simplify your answer)

(c) The 30-year-old should have $___ in education debt.
(Simplify your answer)
.

Answers

The 40-year-old should have $76800 in capital. The 35-year-old should be saving $$10,320 for retirement.

How to solve for the amounts

The capital that a 40 year old has to have with an income of 32000 is

capital income at age 40 * income

= 2.4 * 32000

Capital = $76800

b. At 35 the savings income has to be

percentage of savings income * income

= 12% * $86000

= $10,320

c. There is no specified value for education earnings.

Read more money ratios here:

https://brainly.com/question/16812427

bag contains one red pen, four black pens, and three blue pens. Two pens are randomly chosen from the bag and
are not replaced.
To the nearest hundredth, what is the probability that a black pen IS chosen first and then another black pen is
chosen?

Answers

ANSWER AT THE BOTTOM

Question:

There is a bag with 1 red pen, 4 black pens, and 3 blue pens. If 2 pens are chosen from the bag without replacement, what is the probability that you chose 2 black pens.

Explanation:

Right now there are a total of 8 pens. 4 of them are black. So, the probability of choosing a black pen right now is 4/8, or 1/2.

Lets assume we picked a pen and got black.

Now there are only 7 pens in the bag, and only 3 of them are black. The probability of choosing a black pen right now is 3/7.

So on the first draw, the probability is 1/2

And on the second draw, the probability is 3/7

To find the probability of 2 ocurrences happening in a row, we must multiply their individual probabilities.

For example, if we wanted to find the probability of rolling two 6's in a row on a dice, we would need to mutiply the individual probability together. The probability of rolling one 6 is 1/6, so the probability of rolling two 6's in a row is 1/6 MULTIPLIED BY 1/6, which is 1/36.

The probability of rolling two 6's in a row is 1/36.

Lets apply the same principle to our situation right now.

So on the first draw, the probability is 1/2

And on the second draw, the probability is 3/7

1/2 MULTIPLIED BY 3/7    =    3/14

3/14 in decimal form is 0.21

ANSWER:

0.21, OR 21%

What are the missing parts that correctly complete the proof?


Drag the answers into the boxes to correctly complete the proof.

(Please refer to the images provided for the answer options and equation image.)

Answers

It has been proved that point A is equidistant from the sides of angle PQR.

What is a Bisector ?

Any line or point that divides an angle or a side in equal parts is called a Bisector.

It is given that

Point A is the bisector of Angle PQR

Referring to the image

1. QA is the bisector of angle PQR : Given

2.Angle PQA = Angle RQA : Definition of Bisector

3.Angle QXA = Angle QYA : Definitions of Perpendicular

4. Angle QXA = Angle QYA : All right angles are congruent.

5. QA ≅QA :  Reflexive Property of Congruence

6. ΔPQA ≅ΔRQA : AAS Congruence Postulate

7. AX ≅AY  Corresponding parts of Congruent Triangles

8. Point A is equidistant from the sides of Angle PQR : Definition of equidistant.

To know more about Bisector

https://brainly.com/question/12896755

#SPJ1

Situation:
A 45 gram sample of a substance that's
used to sterilize surgical instruments has
a k-value of 0.15.
N = Noe kt
No initial mass (at time t = 0)
N= mass at time t
k= a positive constant that depends on
the substance itself and on the units
used to measure time
t-time, in days
Find the substance's half-life, in days.
Round your answer to the nearest tenth.
Enter the correct answer.
DONE
+?

Answers

The substance's half-life is 4.7 days if the 11-gram sample of a substance that’s used to treat thyroid disorders has a k- the value of 0.15

What is exponential decay?

During exponential decay, a quantity falls slowly at first before rapidly decreasing. The exponential decay formula is used to calculate population decline and can also be used to calculate half-life.

We have an exponential function:

[tex]\rm N = N_oe^{-kt}[/tex]

Plug N = N⁰/2

[tex]\rm N_o/2 = N_oe^{-kt}[/tex]

[tex]\rm \dfrac{1}{2} = e^{-0.15t}[/tex]

Solving for t:

t = 4.62≈ 4.7 days

Thus, the substance's half-life is 4.7 days if the 11-gram sample of a substance that’s used to treat thyroid disorders has a k- the value of 0.15

Learn more about exponential decay here:

brainly.com/question/14355665

#SPJ1

Un cartero reparte 10.700 cartas al mes. estimar cunats cartas reparte en un semestre

Answers

Teniendo en cuenta la regla de tres simple, el cartero reparte 64.200 cartas en un semestre.

Regla de tres

La regla de tres es una forma de resolver problemas de proporcionalidad entre tres valores conocidos y un valor desconocido, estableciendo una relación de proporcionalidad entre todos ellos.

Es decir, lo que se pretende con ella es encontrar el cuarto término de una proporción conociendo los otros tres.

Si la relación entre las magnitudes es directa, es decir, cuando una magnitud aumenta, la otra también (o cuando una magnitud disminuye, la otra también), se debe aplicar la regla de tres directa.

Para resolver una regla de tres directa se debe seguir la siguiente fórmula, siendo a, b y c datos conocidos y x la variable a calcular:

a ⇒ b

c ⇒ x

Entonces: [tex]x=\frac{cxb}{a}[/tex]

Cartas repartidas en un semestre

En este caso, se puede aplicar la regla de tres de la siguiente manera: Entonces, si el cartero en un mes reparte 10.700 cartas, ¿en 6 meses (un semestre) reparte cuántas cartas?

1 mes ⇒ 10.700 cartas

6 meses ⇒ ×

Entonces: [tex]cantidad de cartas=\frac{6 mesesx10.700 cartas}{1 mes}[/tex]

Resolviendo:

cantidad de cartas= 64.200 cartas

En resumen, el cartero reparte 64.200 cartas en un semestre.

Aprende más con estos ejemplos:

https://brainly.com/question/21685920

https://brainly.com/question/24786153

#SPJ1

Other Questions
Give some examples of what are sand good for explaining is involved! A restaurant offers 7 appetizers and 8 main courses. In how many ways can a person order a two-course meal?There are ways a person can order a two-course meal. A function is a rule thats assigns each value of the independent variable to exactly ____ value of he dependent variable.Free response: Essman might overlook strategic risks, the business plan at hand can be a good plan and the Product mix may be one of the best. However, things do change, and the plans and the product mix stated in the plan may become outdated making the strategy of the company less efficient to reach its goals. Technological risks may also render the product mix out of date. With the dynamism of technology other efficient products may come to the market making rendering the product mix obsolete. Would risk retention be a good strategy for this company? Why or why not? X during the 1780s, most state governments a moved to limit popular power. b remained strongly elitist. c) eliminated property requirements for voters. were notable for their stability. e found it difficult to revise their constitutions. question 5 47 of 50 which matrix equation represents the system of equations Suppose your main competitor has just introduced a new technology by which customers are really delighted about that technology. the customers are increasing substantially after introducing that new technology in the market. as a top-level manager, what should you do at this moment for your organization? An imaginary element, Y, has two different isotopes.y has a percentage abundance of 25% and 29y has a percentage abundance of75%.Which of the following is the relative atomic mass of Y?26252728 Which of the sentences is grammatically correct?The number of men required is large.The number of men required are large.The number of men required has large.The number of men required have large. 4. If u = {1,2,3,....10) A = { odd numbers up to 9} B = {numbers less than 7). Write out the members of the following sets: (i) AUB (ii) An B (ii) A(prime) U B 1.Which thematic group uses technology to direct the behavior of dynamical systems, ensuring that they behave in a predictable manner?2.Which thematic group creates circuits with miniature, packaged components that directs and control electricity?a.Communication Systemsb.Control Systemsc.Computer Systemsd.Energy Systemse.Electronic Systems need help on this ASAP! please What does viscosity describe Dominic had a coupon for 15 off his purchase at Save4You. At DiscountsRUs, there was a sale of 40% off on any purchase. Which store offered the better discount, and how much is the difference? 48 + what gives you 180 Today an electronics store took 5% off the price of a computer, and for the next two days, it will take 5% off the previous day's price. if the price of the computer yesterday was $2000.00, what will be the price of the computer two days from now? 5 Which statement BEST describes the relationship between photosynthesis and cellular respiration?A. Both processes use energy to form molecules of glucose.B. The products of one process are the reactants in the other process.C. The products of one process are the same products in the other process.luminate EducationTM, Inc. A condominium is taxed based on its $78,584 value. The tax rate is $3.49 for every $100 of value. If the tax is paid before March 1, 4% of the normal tax is given as a discount. How much tax is paid if the condominium owner takes advantage of the discount? a. d(7) 6b. Interpret the meaning of d(7):OA. In the first 6 seconds, the particle moved a total of 7 feet.B. The particle was 7 feet away from the starting line 6 seconds since timing started.OC. The particle was 6 feet away from the starting line 7 seconds since timing started.D. In the first 7 seconds, the particle moved a total of 6 feet.c. Solve d(t) = 2 for t. Use commas to separate your answers if there are more than one solution. t =d. Interpret the meaning of part c's solution(s):OA. The article was 2 feet from the starting line 9 seconds since timing started.OB. The article was 2 feet from the starting line 1 seconds since timing started.C. The article was 2 feet from the starting line 1 seconds since timing started, or 9 seconds since timing started.D. The article was 2 feet from the starting line 1 seconds since timing started, and again 9 seconds since timing started.dit on this problem. an economic policy maker would rank a _____ shock as the least preferred type. a. positive demand b. negative demand c. positive supply d. negative supply